You are on page 1of 50

Article XII.

National Economy and Patrimony


Section 2. Regalian Doctrine
Lee Hong Kok v. David, 48 SCRA 372
Lee Hong Hok v. David
Facts:
This is regarding a piece of land which Aniano David acquired lawful title thereto, pursuant to his
miscellaneous sales application. After approval of his application, the Director of Lands issued an
order of award and issuance of sales patent. The Register of Deeds then issued an original
certificate of title to David. Years after petitioners claims that the property was owned by their
predecessor-in-interest thru prescription. Director of lands countered that the land as of public
dominion
Issue:
What is jura regalia?
Held:
The concept of jura regalia means that ownership is vested in the State, not in the head thereof.

IMPERIUM vs. DOMINIUM


Imperium is government authority possessed by the State expressed in the concept of
sovereignty comes.
Dominium is the States capacity to own or acquire property.
Carino v. Insurer Government, 41 PHIL 935
Carino v. Insular Government
Facts:
Mateo Cario (Igorot) filed petition his inscription as the owner of land hes been possessing in
Benguet. Mateo only presented possessory information and no other documentation. The State
opposed the petition averring that the land is part of the US military reservation. Mateo averred
that a grant should be given to him by reason of immemorial use and occupation by his
predecessors-in-interest
Issue:
When the regalian doctrine was introduced into the Philippines by colonizers, did they strip the
natives of their ownership of lands?
Held:
No. When, as far back as testimony or memory goes, the land has been held by individuals
under a claim of private ownership, it will be presumed that to have been held in the same way
from before the Spanish conquest, and never to have been public land.
A native title to land in the Province of Benguet, which for more than 50 years prior to the treaty
of peace with Spain, a native Igorot and his ancestors have held in accordance with Igorot custom,
as private property, should be recognized by the insular government, although no document of
title has issued from the Spanish Crown, without reference to the effect of the change of
sovereignty.
Cruz v. Sec. of DENR, 347 SCRA 128 (2000)
Cruz v. Sec. of DENR
Facts:
Isagani Cruz questions the validity of IPRA law on the grounds of, among others:
Unlawful deprivation of the States ownership over lands of the public domain as well as
minerals and other natural resources therein, in violation of the Regalian doctrine
definition of "ancestral domains" and "ancestral lands" which might even include private
lands found within said areas, Sections 3(a) and 3(b) violate the rights of private
landowners
VOTE: (7-7), after redeliberation, still 7-7. Hence, the petition is DISMISSED.
Held:
The capacity of the State to own or acquire property is the states power of dominium. This was
the foundation for the early Spanish decrees embracing the feudal theory of jura regalia.
Sta. Rosa Mining v. Leido, 156 SCRA 1 (1987)
Sta. Rosa Mining v. Leido
Facts:
PD 1214 was issued requiring holders of subsisting and valid patentable mining claims to file a
mining lease application within 1 year from the approval of the Decree. To protect its rights, Santa
Rosa Mining Company files a r certiorari and questions the constitutionality of the decree as it
amounts to a deprivation of property without due process of law.
Issue:
Is PD1214 constitutional?
Held:
YES. PD 1214 is a valid exercise of the sovereign power of the State, as owner, over lands of the
public domain, of which petitioner's mining claims still form a part, and over the patrimony of
the nation, of which mineral deposits are a valuable asset.
San Miguel Corporation v. CA, 185 SCRA 722 (1990)
San Miguel Corporation v. CA
Fact:
SMC bought a parcel of land from Perez. It sought the registration of the land but was opposed
by SolGen, claiming that the public domain and that SMC and/ or its predecessor-in-interest does
not have a registrable right over the property.
Issue:
Can prescription transform public land into private land?
Held:
YES. Open, exclusive and undisputed possession of alienable public land for the period prescribed
by law creates the legal fiction whereby the land, upon completion of the requisite period ipso
jure and without the need of judicial or other sanction, ceases to be public land and becomes
private property. Such open, continuous, exclusive and notorious occupation of the disputed
properties for more than 30 years must, however, be conclusively established. This quantum of
proof is necessary to avoid the erroneous validation of actually fictitious claims of possession
over the property in dispute
Almeda v. Court of Appeals, GR No. 85322, April 30, 1991
ALMEDA v CA

- In 1984, Alfredo Almeda et al sought to register a parcel of land in Rizal, claiming that
they had been in possession of the lot in a manner that was public, peaceful,
continuous, and adverse to the whole world since 1918, or for a period of more than 30
years
- The trial court granted their petition
- The Republic appealed to the Court of Appeals, on the ground that the subject lot was
released from its forest classification only in 1968, giving the Almedas only 16 years of
possession over it as alienable land
- According to the Republic, their possession of the land before 1968 shold not be
counted, because then it was inalienable land belonging to the public domain
- The CA reversed the lower court and denied the registration

HELD: The Almedas had not qualified for a grant under Section 48(b) of the Public Land Act
because their possession of the land while it was still inalienable forest land, or before it was
declared alienable and disposable land of the public domain in 1968, could not ripen into
private ownership, and should be excluded from the computation of the 30-year open and
continuous possession in concept of owner required under Section 48(b) of Com. Act 141.
Unless and until the land classified as forest is released in an official proclamation to that effect
so that it may form part of the disposable agricultural lands of the public domain, the rules on
confirmation of imperfect title do not apply. Possession of forest lands, however long, cannot
ripen into private ownership
Director of Lands v. Kalahi Investments, Inc, GR No. 48066, January 31, 1989
DIRECTOR OF LANDS v KALAHI INVESTMENTS

- Kalahi Investments seeks to register a parcel of land in Pampanga and Zambales


- Kalahi claims to have perfected mining claims over the subject lot, using as basis the Old
Mining Law, and the opinion of the Secretary of Justice in 1956, which states thus:

The legal effect of a valid location of a mining claim is not only to segregate the area
from the public domain, but to grant to the locator the beneficial ownership of the claim
and the right to a patent therefore upon compliance with the terms and conditions
prescribed by law. Where there is a valid location of the mining claim, the area becomes
segregated from the public domain and becomes the property of the locator.

Issue: Does mere location of mining claims mature to private ownership that would entitle the
claimant to the ownership thereof?

HELD: NO. Mere location does not mean absolute ownership over the affected land or the
located claim. It merely segregates the located land or area from the public domain by barring
other would-be locators from locating the same and appropriating for themselves the minerals
found therein. To rule otherwise would imply that location is all that is needed to acquire and
maintain rights over a located mining claim.
Republic v. Enciso, GR No. 160145, November 11, 2005
REPUBLIC v ENCISO

- Pedro Enciso filed a petition for land registration before the RTC of Iba, Zambales
- Enciso claimed that the subject lot was originally the property of the Municipality of
Masinloc, and was reclaimed land
- In the process of building a shoreline road, lot owners were asked to give up part of
their land; in exchange the Municipality of Masinloc sold to these lot owners reclaimed
land
- One of the buyers of the reclaimed land was Honorato Edao, the half-brother of Vicente
Enciso (Pedros father)
- Honorato sold the lot to Vicente, and Pedro inherited it when Vicente died
- The RTC approved the petition for registration
- The Republic appealed to the CA, which affirmed the RTC decision; the Republic argued
that Enciso failed to show that the land was part of alienable and disposable land of
public domain

ISSUE: W/N Enciso should be allowed to register the land given that it is reclaimed

HELD: NO. The land was inalienable.


For reclaimed land to be registered as private property, the claimant must prove the following:

1) The land had been classified as alienable, since reclaimed land is generally part of the
inalienable public domain; and
2) That he (the claimant) acquired the property through some means, e.g. prescription
However, it is important to note that inalienable land cannot be acquired by prescription, so
since Enciso failed to prove the first element above, no period of time can result in private
ownership over the subject land.
Chavez v. PEA, GR No. 133250, July 9, 2002
CHAVEZ v PEA

President Marcos through a presidential decree created PEA, tasked with the developmen
t, improvement, and acquisition, lease, and sale of all kinds of lands. He also transferred to
PEA the foreshore and offshore lands of Manila Bay under the Manila-Cavite Coastal Road and
Reclamation Project.

Thereafter, PEA was granted patent to the reclaimed areas of land and then, years later,
PEA entered into a JVA with AMARI for the construction of the Freedom Islands project.

Later, a privilege speech was given by Senate President Maceda denouncing the JVA as the
grandmother of all scams. An investigation was conducted and it was concluded that the lands
that PEA was conveying to AMARI were lands of the public domain.
ISSUE: W/N stipulations in the amended JVA for the transfer to AMARI of the lands,
reclaimed or to be reclaimed, violate the Constitution.

HELD: YES. The ownership of lands reclaimed from foreshore and submerged areas is rooted in
the Regalian doctrine, which holds that the State owns all lands and waters of the public
domain.

The 1987 Constitution recognizes the Regalian doctrine. It declares that all
natural resources are owned by the State and except for alienable
agricultural lands of the public domain, natural resources cannot be alienated.

The foreshore and submerged areas of Manila Bay are part of the lands of
the public domain, waters and other natural resources and consequently owned by the
State. As such, foreshore and submerged areas shall not be
alienable unless they are classified as agricultural lands of the public domain. The mere
reclamation of these areas by the PEA doesnt convert
these inalienable natural resources of the State into alienable and disposable lands of the
public domain. There must be an act of
Congress officially classifying these reclaimed lands as alienable and disposable.

Likewise, any legislative authority given to PEA to sell the reclaimed lands should only benefit
private individuals, and not corporations, as the Constitution prohibits corporations from
acquiring alienable lands.
Laurel v. Garcia, 187 SCRA 797 (1990)
LAUREL v GARCIA

FACTS: The subject Roppongi property is one of the properties acquired by the Philippines from
Japan pursuant to a Reparations Agreement. The property is where the Philippine Embassy was
once located, before it transferred to the Nampaidai property. It was decided that
the properties would be
available to sale or disposition. One of the first properties opened up for public auction was the
Roppongi property, despite numerous oppositions from different sectors.

ISSUE: May the Roppongi property be the subject of alienation?

HELD: NO. It is of public dominion unless it is convincingly shown that the property has become
patrimonial. The respondents have failed to do so.
As property of public dominion, the Roppongi lot is outside the commerce of man. It cannot be
alienated. Its ownership is a special collective ownership for general use and payment, in
application to the satisfaction of collective needs, and resides in the social group. The
purpose is not to serve the State as the juridical person but the citizens; it is intended for the
common and public welfare and cannot be the object of appropriation.

The fact that the Roppongi site has not been used for a long time for actual Embassy service
doesnt automatically convert it to patrimonial property. Any such conversion happens only if
the property is withdrawn from public use. A property continues to be part of the public
domain, not available for
private appropriation or ownership until there is a formal declaration on the part of the
government to withdraw it from being such.
Miners Association v. Factoran, 240 SCRA 100 (1995)
MINERS ASSOCIATION vs. FACTORAN

EO 211 and 279, issued by Pres. Corazon Aquino, authorizes the DENR Sec to enter into
joint-venture, co-production, or production- sharing agreements for the exploration,
development, and utilization of mineral resources.
By virtue of which the DENR issued:
o AO 57 which provides that all existing mining leases or agreements which were
granted after the effectivity of the 1987 Constitutionshall be converted into
production-sharing agreements.
o AO 82 which provides that a failure to submit Letter of Intent and Mineral
Production-Sharing Agreement shall cause the abandonment of the mining,
quarry, and sand and gravel claims, after their respective effectivity dates.
Petitioner Miners Association now questions the validity of the two AOs.

ISSUE: WON the two Department Administrative Orders valid?


RULING: YES. The State, in the exercise of its police power in this regard, may not be precluded
by the constitutional restriction on non-impairment of contract from altering, modifying and
amending the mining leases or agreements granted under Presidential Decree No. 463, as
amended, pursuant to Executive Order No. 211. Police Power, being co-extensive with the
necessities of the case and the demands of public interest; extends to all the vital public needs.
The passage of Executive Order No. 279 which superseded Executive Order No. 211 provided
legal basis for the DENR Secretary to carry into effect the mandate of Article XII, Section 2 of the
1987 Constitution.
Republic v. Rosemoor, GR No. 149927, March 30, 2004
REPUBLIC vs. ROSEMOOR

Dr Pascual and his group, after having granted permission to search for marble deposits
in Biak-na-Bato, Bulacan, succeeded in discovering marble deposits.
They then applied in with the Bureau of Mines(now MGB) for the issuance of license to
exploit said marbles, which was later on issued to them.
Maceda, the new minister of DENR, thereafter cancelled the said license.

ISSUE: WON a license granted by the government can be withdrawn or revoked?


RULING: The revocation is justified as the respondents license may be revoked or rescinded by
executive action when the national interest so requires, because it is not a contract, property or
a property right protected by the due process clause of the Constitution. As all lands of the
public domain belong to the State, the state may curtail its use or exploitation if the national
interest so requires.
La Bugal-Blaan Tribal Assn. v. DENR, GR127872, Jan 27, 2004, MR GR 127882, Dec. 1, 2004
LA BUGAL-B'LAAN vs DENR

On January 27, 2004, the Court en banc promulgated its Decision granting the Petition
and declaring the unconstitutionality of certain provisions of RA 7942, DAO 96-40, as
well as of the entire FTAA executed between the government and WMCP, mainly on the
finding that FTAAs are service contracts prohibited by the 1987 Constitution.
The Decision struck down the subject FTAA for being similar to service contracts, which,
though permitted under the 1973 Constitution, were subsequently denounced for being
antithetical to the principle of sovereignty over our natural resources, because they
allowed foreign control over the exploitation of our natural resources, to the prejudice
of the Filipino nation.

ISSUE: Are foreign-owned corporations in the large-scale exploration of our natural resources
limited to technical or financial assistance only?
RULING: Only technical assistance or financial assistance agreements may be entered into, and
only for large-scale activities. Full control is not against the day-to-day management by the
contractor, provided that the State retains the power to direct overall strategy; and to set
aside, reverse or modify plans and actions of the contractor. The idea of full control is similar
to that which is exercised by the board of directors of a private corporation: the performance of
managerial, operational, financial, marketing and other functions may be delegated to
subordinate officers or given to contractual entities, but the board retains full residual control
of the business.
Philippine Geothermal v. Napocor, GR No. 144302, May 27, 2004
PHIL. GEOTHERMAL INC. vs. NAPOCOR

NPC entered into a service contract with PGI (US Corp.) for exploitation of geothermal
resources covering Tiwi and Mak-ban geothermal fields.
One of the provisions in the contract was The term of this contract shall be twenty-five
(25) years renewable for another twenty-five (25) years upon the option of PGI under
the same terms and conditions set forth herein.
The service contract was to expire but NPC was doubtful whether a renewal would be
constitutional in light of Section 2, Article XII of the 1987 Constitution.

ISSUE: (ONLY PROCEDURAL MATTERS)


RULING: It was ruled that the lower courts decisions were null and void as the parties are still
undergoing arbitration.
Both parties agreed to terminate the Service Contract subject matter of the dispute, in favor of
a new Geothermal Sales Contract and a PD 1442 Geothermal Service Contract, and PGI has
committed to form a Philippine company for the development and operation of the Tiwi and
Mak-Ban steamfields on a going-forward basis, thereby effectively erasing any doubt as to the
legality of the compromise.
JG Summit v. CA, GR No. 124293, January 31, 2005
Republic v. Domingo Espinosa, G.R. No. 176885, July 5, 2010
Republic v. Domingo Espinosa, G.R. No. 176885, July 5, 2010
Facts:

Respondent filed an application for the registration of a certain lot. He claims that:
He acquired the land from his mother.
He has been in open, public, continuous and notorious possession of the land in
the concept of an owner for more than thirty (30) years.
He had also verified in a notation on the right side portion of the plan that the lot
is within the alienable and disposable area.
The Petitioners filed an appeal to the ruling of the MTC that granted the registration and
claims that the respondent failed to prove that the land has been classified as alienable
or disposable.
Issue: Whether the land has been classified as alienable or disposable?
Ruling: NO. A review of the records shows that other than the notation on the advanced survey
plan stating in effect that the subject property is alienable and disposable and respondents self-
serving testimony, there is an utter lack of evidence to show the actual legal classification of the
disputed lot. Respondent was not able to show proof that the property was alienable or
disposable.
The classification and reclassification of public lands into alienable or disposable, mineral or
forest land is the prerogative of the Executive Department. Under the Regalian doctrine, which
is embodied in our Constitution, all lands of the public domain belong to the State. All lands not
appearing to be clearly within private ownership are presumed to belong to the State.
Accordingly, public lands not shown to have been reclassified or released as alienable agricultural
land or alienated to a private person by the State remain part of the inalienable public domain.
Hontiveros-Baraquel v. Toll Regulatory Board, G.R. No. 181293, February 23, 2015
Hontiveros-Baraquel v. Toll Regulatory Board, G.R. No. 181293, February 23, 2015
Facts:

The Philippine National Construction Corporation (PNCC), pursuant to P.D. 1113 with the
right, privilege, and authority to construct, and operate toll facilities Toll expressways, in
a series of agreements transferred authority to perform operations of the South Metro
Manila Highway to Skyway O & M Corporation (SOMCO). The Legislators and the Union
of PNCC oppose the said transfer. They argue that the Toll & operation Certificate issued
by the the Toll Regulatory board (TRB) to SOMCO is highly irregular and that the transfer
of authority is grossly disadvantageous to the government.
One of the contention of the Petitioners is that SOMCO is not qualified to operate a toll
facility, because it does not meet the nationality requirement for a corporation. They
contend that 40% of SOMCO is owned by CMMTC, a foreign company.
Issue: Whether SOMCO is not qualified to operate because it does not meet the nationality
requirement?
Ruling: NO. Petitioners have not shown how SOMCO fails to meet the nationality requirement
for a public utility operator. It is axiomatic that one who alleges a fact has the burden of proving
it. On this matter, the court found that petitioners have failed to prove their allegation that
SOMCO is not qualified to operate a toll facility for failure to meet the nationality requirement
under the Constitution.
Spouses Antonio and Erlinda Fortuna v. Republic, G.R. No.173423, March 5, 2014
Spouses Antonio and Erlinda Fortuna v. Republic, G.R. No.173423, March 5, 2014
Facts:

Petitioners filed an application of a lot claiming that through themselves and their
predecessors-in-interest, have been in quiet, peaceful, adverse and uninterrupted
possession of the lot for more than 50 years, and submitted as evidence the lots survey
plan, technical description, and certificate of assessment.
The Republic claims that the petitioners did not present an official proclamation from the
government that the lot has been classified as alienable and disposable agricultural land.
Issue: Whether the disputed lot has been classified as alienable and disposable agricultural land?
Ruling: NO. Public land that has not been classified as alienable agricultural land remains part of
the inalienable public domain. This right to classify is vested to the executive department. Thus,
it is essential for any applicant for registration of title to land derived through a public grant to
establish foremost the alienable and disposable nature of the land.
To be granted, they must be grounded in well-nigh incontrovertible evidence. Where, as in this
case, no such proof would be forthcoming, there is no justification for viewing such claim with
favor. It is a basic assumption of our polity that lands of whatever classification belong to the
state. Unless alienated in accordance with law, it retains its rights over the same as do minus.
Gaerlan v. Republic, G.R. No. 192717, March 12, 2014
Gaerlan v. Republic, G.R. No. 192717, March 12, 2014
Facts:

Petitioner applied for an application of title for a parcel of land that she acquired by virtue
of a Deed of Absolute Sale of Unregistered Land and presented the following documents:
(a) Original Tracing Cloth Plan together with the three (3) Blue print copies
(b) Technical Description of the parcel of land;
(c) Surveyor's Report of Survey or Surveyor' s Certificate;
(d) Deed of Absolute Sale of Unregistered Land.
(e) Tax Declaration No. 99893.
The Republic filed an opposition claiming that the subject land is a portion of the public
domain, hence, not registrable.
Issue: Whether the disputed land is alienable?
Ruling: NO. Under the Regalian doctrine, all lands of the public domain belong to the State. The
burden of proof in overcoming the presumption of State ownership of the lands of the public
domain is on the person applying for registration, who must prove that the land subject of the
application is alienable and disposable. To overcome this presumption, incontrovertible evidence
must be presented to establish that the land subject of the application is alienable and
disposable.
Petitioner failed to prove that:
(1) the subject property was classified as part of the disposable and alienable land of the public
domain; (2) she and her predecessors-in-interest have been in open, continuous, exclusive, and
notorious possession and occupation thereof under a bona fide claim of ownership since June
12, 1945 or earlier.
Peza v. Carantes, G.R. 181274, June 23, 2010
Peza v. Carantes, G.R. 181274, June 23, 2010
Facts:

Respondents was in possession of a parcel of land where they fenced the premises and
constructed a residential building thereon. Respondents received a letter from Philippine
Economic Zone Authority (PEZA) informing them that the house they built had overlapped
PEZAs territorial boundary and told them to demolish the same.
Without answering PEZAs letter, respondents filed a petition for injunction, with prayer
for the issuance of a temporary restraining order (TRO) and writ of preliminary injunction
which was granted by the court. The court further ruled that that respondents are entitled
to possess, occupy and cultivate the subject lots on the basis of their Certificate of
Ancestral Land Claim (CALC).
Issue: Whether the respondents can build a fence and house on the disputed land?
Ruling: NO. Respondents being holders of a mere CALC, their right to possess the subject land is
limited to occupation in relation to cultivation. Even if respondents had established ownership of
the land, they cannot simply put up fences or build structures thereon without complying with
applicable laws, rules and regulations.
PEZA acted well within its functions when it demanded the demolition of the structures which
respondents had put up without first securing building and fencing permits from the Authority.
Section 3. Lands of the Public Domain
Director of Lands v. Aquino, 192 SCRA 296 (1990)

Director of Lands v. Aquino, 192 SCRA 296 (1990)

FACTS: This case involves a limestone-rich land in Abra of which are, according to
petitioners, within the Central Cordillera Forest Reserve.
ISSUE: WON the land was alienable?

HELD: The fact that the contested parcels of land have long been denuded and
actually contains rich limestone deposits does not in any way affect its present
classification as forest land.
The change the location of a communal forest, such executive action does not
amount to a declassification of a forest reserve into an alienable or disposable
land.: n
That forest lands or forest reserves are incapable of private appropriation and
possession thereof, however long, cannot convert them into private properties.
This ruling is premised on the Regalian doctrine enshrined not only in the 1935 and
1973 Constitutions but also in the 1987 Constitution Article XIII of which provides
that:
"Sec. 2. All lands of the public domain, waters, minerals, coal . . . , forests or timber,
. . . and other natural resources are owned by the State. With the exception of
agricultural lands, all other natural resources shall not be alienated."
The land must first be released from its classification as forest land and reclassified
as agricultural land because the classification of public lands is an exclusive
prerogative of the executive department of the government and not of the
courts.
Republic v. CA, 160 SCRA 228 (1988)
Apex Mining v. Southeast Mindanao Gold, Inc, GR No. 152613, June 23, 2006
Dir. of Lands v. IAC, 146 SCRA 509 (1986)
Ten Forty Realty v. Lorenzana, GR No. 151212, Sept. 10, 2003
Chavez v. PEA, GR No. 133250, July 9, 2002
Section 3. Lands of the Public Domain
Francisco Chavez v. Public Estates Authority, Amari Coastal Bay Development Corp (GR
No. 133250, July 9, 2002)
FACTS:
PEA (created in 1977 by then-President Marcos), as the government agency tasked with
reclaiming lands was able to acquire the Freedom Islands thru a Special Patent issued
by then-President Aquino
o The Freedom Islands is a 157-hectare part of the reclaimed land in the Manila-
Cavite Coastal Road and Reclamation Project of the government and a private
company
Later, in 1995, PEA entered into a Joint Venture Agreement (JVA) with private
respondent Amari to develop the Freedom Islands (without bidding) plus the reclamation
of another 250 hectares which then-President Ramos approved
o However, the JVA was questioned by the Senate because the land covered by
such was inalienable
Nevertheless, in 1998, PEA and Amari renegotiated the JVA prompting petitioner
Chavez, as taxpayer, to institute this present action on the ground that the transfer of
these lands to Amari will cause the government to lose billions of pesos
o He argues that the lands are part of the public domain and thus inalienable
o Further, he seeks the disclosure of all terms of the renegotiation
The renegotiation resulted to an Amended JVA which was approved by then-President
Estrada in 1999
ISSUE:
Whether the reclaimed land/about-to-be-reclaimed land may be sold to Amari
RULING:
As to the 157-hectare reclaimed land, NO
o Although such lands are already part of the alienable lands of public domain, PEA
is only allowed to lease it to private corporations f or a period not exceeding 25
years, renewable for not more than 25 years, and not to exceed 1,000 hectares
o PEA may only sell it Filipino citizens subject to Constitutional limitations (12
hectares only by purchase, homestead or grant)
As to the about-to-reclaimed land, NO (they are inalienable and outside the commerce of
men)
o The 592.15 hectares of submerged areas of Manila Bay remain inalienable natural
resources of the public domain until classified as alienable or disposable lands
open to disposition and declared no longer needed for public service

Section 4. Specific Limits of Forest Lands and National Parks


La Bugal-Blaan Tribal Assn. v. DENR, GR127872, Jan 27, 2004, MR GR 127882, Dec. 1, 2004
Section 4. Specific Limits of Forest Lands and National Parks
La Bugal-Blaan Tribal Association, et al v. DENR Secretary, MGB Director, Executive
Secretary, WMC Philippines Inc (GR 127872, Jan 27, 2004)
FACTS:
This petition challenges the constitutionality of RA 7942 (The Philippine Mining Act of
1995), its IRR, and the Financial and Technical Assistance Agreement (FTAA) dated
March 30, 1995 by the government with Western Mining Corporation (Philippines) Inc.
(WMCP)
o In that agreement, 99,387 hectares of land in South Cotabato, Sultan Kudarat,
Davao del Sur and North Cotabato shall be covered in favor of WMCP
o Furthermore, they alleged that, at the time of filing of this petition, over 100
FTAAs have been already filed, 64 of which are by fully foreign-owned
corporations
Accordingly, the FTAA violated the 1987 Constitution in that it is a service contract and
is antithetical to the principle of sovereignty over our natural resources, because they
allowed foreign control over the exploitation of our natural resources, to the prejudice of
the Filipino nation
o Initially, WMCP was a wholly-owned subsidiary of WMC LIMITED, an
Australian mining company, but was later sold to Sagittarius Mining, a local
corporation (60-40 owned)
o Nevertheless, the validity of transfer of ownership was then-pending at the time of
this petition
ISSUE:
Whether RA 7942, its IRR and the FTAA are unconstitutional
RULING:
YES, the constitutional provision allowing the President to enter into FTAA is an
exception to the rule that participation in the nations natural resources is reserved
exclusively to Filipinos
o Provision must be construed strictly against their enjoyment by non-Filipinos
The FTAA is, in a reality, a service contract which grant WMCP beneficial ownership
over natural resources that properly belong to the State and are intended for the benefit of
its citizens

MR GR 127882, Dec. 1, 2004


RULING:
NO, RA 7942, its IRR and the FTAA are constitutional
o Full control is not anathematic to day-to-day management by the contractor,
provided that the State retains the power to direct overall strategy; and to set
aside, reverse or modify plans and actions of the contractor
The idea of full control is similar to that which is exercised by the board of directors of a
private corporation, the performance of managerial, operational, financial, marketing and
other functions may be delegated to subordinate officers or given to contractual entities,
but the board retains full residual control of the business

Section 5. Ancestral Lands and Domain


Cruz v. Sec. of DENR, 347 SCRA 128 (2000)
Section 5. Ancestral Lands and Domain
Isagani Cruz, Cesar Europa v. Sec. of DENR (GR 135385, December 6, 2000)
FACTS:
Petitioners, as citizens and taxpayers, are assailing the constitutionality of certain IPRA
Law (RA 8371) provisions and its IRR
o GROUND:
Violation of Regalian Doctrine unlawful deprivation of States
ownership over lands of public domain (including natural resources &
minerals)
Violation of rights of private landowners private lands located inside
ancestral lands/domains
Infringement on the Presidents power of control declaring National
Commission on Indigenous People as autonomous
Violation of due process clause
o ASSAILED PROVISIONS
Extent and coverage of ancestral land/domain
Priority rights of IPs to ancestral land/domain
Exclusive power of NCIP over ancestral land/domain; Lateral relationship
of NCIP to the Office of the President
Use of customary law in ancestral land/domain
Upon deliberation and re-deliberation, the Court voted 7-7
ISSUE:
Whether IPRA Law is unconstitutional for allowing IPs to take over natural resources in
their ancestral lands/domains
RULING:
NO, there is nothing in the law that grants to the ICCs/IPs ownership over the natural
resources within their ancestral domain
o Ownership over the natural resources in the ancestral domains remains with the
State and the rights granted by the IPRA to the ICCs/IPs over the natural
resources in their ancestral domains merely gives them, as owners and occupants
of the land on which the resources are found, the right to the small scale
utilization of these resources, and at the same time, a priority in their large scale
development and exploitation.
Additionally, ancestral lands and ancestral domains are not part of the lands of the public
domain
o They are private lands and belong to the ICCs/IPs by native title, which is a
concept of private land title that existed irrespective of any royal grant from the
State
o However, the right of ownership and possession by the ICCs/IPs of their ancestral
domains is a limited form of ownership and does not include the right to alienate
the same

Section 6. Common Good


Telecommunications and Broadcast Attorneys of the Philippines Inc. (TELEBAP), GMA
Network Inc. v. COMELEC (GR 132922, April 21, 1998)
FACTS:
Petitioners TELEBAP (lawyers of radio and TV companies) and GMA (radio and TV
network) assail the constitutionality of Sec 92 (Election Code), in relation to Sec 11.b
(RA 6646)
o Said provisions prohibit the sale or donation of print space or airtime for political
ads, except to the COMELEC
According to petitioners, the assailed provisions are invalid on the grounds:
o (1) that it takes property without due process of law and without just
compensation;
o (2) that it denies radio and television broadcast companies the equal protection of
the laws; and
o (3) that it is in excess of the power given to the COMELEC to supervise or
regulate the operation of media of communication or information during the
period of election.
ISSUE:
Whether GMA may resist the mandate of the assailed provisions by using its franchise to
shield itself
RULING:
NO, A franchise is a privilege subject, among other things, to amendment by Congress in
accordance with the constitutional provision that any such franchise or right granted . . .
shall be subject to amendment, alteration or repeal by the Congress when the common
good so requires

Section 7. Private Lands


Republic v. CA, 235 SCRA 567
Zaragosa v. CA, GR No. 106401, September 29, 2000
Zaragosa v. CA, GR No. 106401, September 29, 2000
FACTS:

Flavio Zaragoza Cano was the registered owner of certain parcels of land in Iloilo. He had
four children: Gloria, Zacariaz, Florentino and Alberta
Later, he died without a will and was survived by his four children.
Alberta filed a complaint against her brother Florentino for delivery of her inheritance
share
She alleged that the shares of her brothers and sister were given to them in advance by
way of deed of sale, but without valid consideration, during their fathers lifetime
She averred that because of her marriage, she became an American citizen. For this
reason, no formal deed of conveyance was executed in her favor covering these lots
during her father's lifetime.
ISSUE:

Was the partition valid?


o YES. A partition inter vivos may be done for as long as legitimes are not
prejudiced. The legitime of compulsory heirs is determined after collation.
Unfortunately, collation can not be done in this case where the original petition
for delivery of inheritance share only impleaded one of the other compulsory
heirs.
Whether Alberta may acquire such land
RULING:

YES. A natural born citizen of Philippines who has lost his/her Philippine citizenship may
be a transferee of private lands or may inherit (through hereditary succession) such
lands.

Ramirez v. Vda. De Ramirez, 111 SCRA 704 (1982)


Ramirez v. Vda. De Ramirez, 111 SCRA 704 (1982)
FACTS:

This case is about the partitioning the testate estate of Jose Eugenio Ramirez (Filipino
who died in Spain) among: his widow Marcelle (French); his two grandnephews Roberto
and Jorge; and his companion Wanda (Austrian).
o Jose willed usufructuary rights over real property to Wanda who is an Austrian
Roberto and Jorge argues that the grant of a usufruct over real property in the
Philippines in favor of Wanda, who is an alien, violates the Constitution
ISSUE:

Whether the usufruct in favor of an alien violates the Constitution


RULING:

NO. Usufruct, albeit a real right, does not vest title to the land in the usufructuary and it
is the vesting of title to land in favor of aliens which is proscribed by the Constitution.

Halili v. CA, 287 SCRA 465 (1998)


Halili vs. CA
FACTS:
Simeon De Guzman, an American citizen, died leaving real properties in Philippines. His
forced heirs were his widow Helen and his son David, both American citizens.
o Helen executed a quitclaim transferring her rights in the lands in favor of David.
o David sold one of the lands to Emiliano, a Filipino citizen.
Petitioners, who are owners of the adjoining lot, questions the constitutionality of the 2
conveyances, and petitioners claimed ownership thereto based on their right of
redemption
ISSUE:
Whether the petitioners have a right to redeem said lot
o NO. The land is urban, thus no right of redemption
Whether the sale to Emiliano is valid
RULING:
YES. If a land is invalidly transferred to an alien who subsequently becomes a citizen or
transfers it to a citizen, the flaw in the original transaction is considered cured and the
title of the transferee is rendered valid. The objective of the constitutional provision to
keep our land in Filipino hands has been served.
The transfer of an interest in a piece of land to an alien may no longer be assailed on
constitutional grounds after the entire parcel has been sold to a qualified citizen.

Lee v. Republic, 366 SCRA (2001)


Lee vs. Republic
FACTS:
The Dinglasans sold a parcel of land to Lee, a Chinese citizen.
Later, the former owners filed an action against the heirs of Lee for annulment of sale.
They assailed that the sale was invalid because of the constitutional prohibition against
aliens acquiring ownership of private agricultural lands
Later, the heirs (Filipino citizens) of Lee filed a petition for reconstitution of title since
the records of the Register of Deeds were burned. This was granted.
More than 60 years later, the Solicitor General initiated an action for annulment of
judgment of the reconstitution of title.
ISSUE:
Whether Lee has the qualification to own the land in the Philippines
RULING:
NO. Aliens could not acquire private agricultural lands, except in cases of hereditary
succession. Thus, Lee, a Chinese citizen, was disqualified to acquire the land in question.
o In sales of real property to aliens, both vendor and vendee are deemed to in pari
delicto. Thus the courts will not afford protection to either party. The proper
party to assail the sale is the Solicitor General.
However, since the said land is now in the hands of Filipinos, it cannot be impugned
on the basis of invalidity of the initial transfer.
o If a land is invalidly transferred to an alien who subsequently becomes a citizen
or transfers it to a citizen, the flaw in the original transaction is considered cured
and the title of the transferee is rendered valid

Frenzel v. Catito, GR No. 143958, July 11, 2003


Frenzel vs. Catito
FACTS:
Alfred Frenzel is an Australian citizen of German descent. He married a Filipino citizen
but they later separated without divorce.
Frenzel met Ederlina Catito, a Filipina who unknown to him was already married to a
Klaus, German national.
o Frenzel was enamored with Catito. He planned to marry Catito. They lived
together. Frenzel offered to finance Catito's business venture in the Philippines.
Frenzel bought several properties in the Philippines but these were all registered under
the name of Catito since Frenzel knew that as an alien he was disqualified from owning
lands in the Philippines
Later the relationship of Frenzel and Catito went sour because Catito was unable to
obtain a divorce with Klaus.
Hence, Frenzel filed against Catito for the recovery of real and personal properties
which were under the latter's name, since he solely acquired such through his own
efforts and resources
ISSUE:
Whether Frenzel is entitled to compensation for the properties
RULING:
NO. Even if Frenzel is the real vendee, the said transactions are in violation of the
Constitution, hence null and void. One who loses his money or property by knowingly
engaging in a contract or transaction which involves his own moral turpitude may not
maintain an action for his loses.
To allow Frenzel to recover would be subversive of public policy.

Mulller v. Muller, GR No. 149615, August 29, 2006


Muller v. Muller
Facts:
Elena Buenaventura (Filipino) married Helmut Muller (German). They resided in Germany but
decided to move and reside permanently in the Philippines. Helmut sold the house he inherited
in Germany to buy a property in Antipolo. The property was registered in Elenas name. Due to
incompatibilities and respondents alleged womanizing, drinking, and maltreatment, the spouses
eventually separated. Hence, they filed for separation of properties. RTC ruled that the properties
within the Philippines shall be divided equally. Except for the property in question, which shall
belong to Elene due to prohibition on alien ownership and that Helmut is not entitled for
reimbursement.
Issue:
Is Helmut entitled for reimbursement?
Held:
NO. He willingly and knowingly bought the property despite a constitutional prohibition. And to
get away with that constitutional prohibition, he put the property under the name of his Filipina
wife. Thus, he cannot seek reimbursement on the ground of equity.
Aliens, whether individuals or corporations, are disqualified from acquiring lands of the public
domain. Hence, they are also disqualified from acquiring private lands. The primary purpose of
the constitutional provision is the conservation of the national patrimony. Except in the case of
hereditary succession, an aliens disqualification from owning lands in the Philippines is
ABSOLUTE, not even an ownership in trust is allowed.
Matthews v. Taylor Spouses, GR No. 164584, June 22, 2009
Matthews v. Taylor Spouses
Facts:
Benjamin (British) married Joselyn (Filipina). During their marriage, Joselyn bough a property in
Boracay and constructed a resort, which was allegedly financed by Benjamin. They separated and
Jocelyn ran away with Philippsen. She then entered into an Lease Agreement with Mathews over
the property in Boracay. Benjamin moved for the nullity of such agreement since he did not
consent to the same.
Issue:
Whether Bejamins consent is necessary for the validity of the Lease Agreement between Jocelyn
and Mathew?
Held:
NO. An alien, since he has no right to acquire private land, has no right to challenge the validity
of the lease over the property which his wife had acquired.
Hulst v. PR Builders, GR No. 156364, September 25, 2008
Hulst v. PR Builders
Fact:
Petitioner spouses are Dutch nationals, they purchased a residential unit in a townhouse project
in Batangas. They paid but PR Builders failed to complete the project. Hence, they filed for
rescission of the contract plus damages.
Issue:
Whether aliens are allowed to own a real estate in the Philippines.
Held:
Aliens can own Philippine real estate through the purchase of condominium units or townhouses
constituted under the Condominium principle. The prohibition covers only land. In cases of
condominium, the land on which the condominium stands is owned by the corporation.
Section 8. Exception for Former Filipino Citizens
Republic v. CA, 235 SCRA 567 (1994)
Section 9. Independent Economic and Planning Agency
Section 10. Filipinization
Manila Prince Hotel v. GSIS
Facts:
Pursuant to the privatization program, GSIS sold most of its shares in Manila Hotel (MHC). In a
closed bidding, only two bidders participated: Manila Prince Hotel (MPHC) and Renong Berhad
Malysian Firm. Several letters were send, matching the bid of the other. In end GSIS decided to
sell the shares to Renong Berhad. MPHC filed a prohibition and mandamus, contending that such
sale would run contrary to Section 10 of Article XII, claiming that qualified Filipino corporation
shall be given preference.
Issue:
Whether Article XII, Section 10 is self-executing.
Held:
Generally, YES. A provision which lays down a general principle is usually not self-executing. But
a provision which is complete in itself and becomes operative without the aid of supplementary
or enabling legislation, or that which supplies sufficient rule by means of which the right it
grants may be enjoyed or protected, is self-executing. Thus a constitutional provision is self-
executing if the nature and extent of the right conferred and the liability imposed are fixed by
the constitution itself, so that they can be determined by an examination and construction of its
terms, and there is no language indicating that the subject is referred to the legislature for action.
Army and Navy Club v. CA, 271 SCRA 36 (1997)
Tanada v. Angara, 272 SCRA 18 (1997)
TANADA v ANGARA
This is a case petition by Sen. Wigberto Tanada, together with other lawmakers, taxpayers, and
various NGOs to nullify the Philippine ratification of the World Trade Organization (WTO)
Agreement.
Petitioners believe that this will be detrimental to the growth of our National Economy and
against to the Filipino First policy. The WTO opens access to foreign markets, especially its
major trading partners, through the reduction of tariffs on its exports, particularly agricultural
and industrial products. Thus, provides new opportunities for the service sector cost and
uncertainty associated with exporting and more investment in the country. These are the
predicted benefits as reflected in the agreement and as viewed by the signatory Senators, a
free market espoused by WTO.

ISSUE: Whether or not the provisions of the Agreement Establishing the World Trade
Organization directly contravene or undermine the letter, spirit and intent of Sections 10
Article XII of the 1987 Constitution.

HELD: Although the Constitution mandates to develop a self-reliant and independent national
economy controlled by Filipinos, does not necessarily rule out the entry of foreign investments,
goods and services. It contemplates neither economic seclusion nor mendicancy in the
international community. The WTO itself has some built-in advantages to protect weak and
developing economies, which comprise the vast majority of its members. Unlike in the UN
where major states have permanent seats and veto powers in the Security Council, in the WTO,
decisions are made on the basis of sovereign equality, with each members vote equal in weight
to that of any other. Hence, poor countries can protect their common interests more effectively
through the WTO than through one-on-one negotiations with developed countries. Within the
WTO, developing countries can form powerful blocs to push their economic agenda more
decisively than outside the Organization. Which is not merely a matter of practical alliances but
a negotiating strategy rooted in law. Thus, the basic principles underlying the WTO Agreement
recognize the need of developing countries like the Philippines to share in the growth in
international trade commensurate with the needs of their economic development.

Section 11. Public Utilities

Bagatsing v. Committee, 246 SCRA 344 (1995)

BAGATSING v COMMITTEE
During the presidency of Corazon Aquino, Petron was a wholly-owned subsidiary of the
Philippine National Oil Corp., and served as the oil refining company of the government.
Consistent with the thrust for the privatization of state-owned enterprises from the Marcos
presidency, the subsequent administration, that of Fidel Ramos, approved the privatization of
Petron. One of the major bidders for the purchase of more than 40% of shares of Petron as
Aramco, a company registered in Saudi Arabia. Aramco was eventually declared the winner of
the bid for the bulk of shares in Petron.
ISSUE: W/N Petron is a public utility, which would mean that the bidding won by Aramco is void
HELD: NO. Accdg to the Constitution and the Public Service Law, a public utility is one organized
for hire or compensation. Since Petron does not engage in oil refining for hire or
compensation, it is not a public utility.
Albano v. Reyes, 175 SCRA 36 (1997)
Albano vs Reyes
Facts: In 1987, the Phil. Ports Authority adopted a resolution to prepare for the public bidding
of the development, mgmt., and operation of the Manila Intl. Container Terminal. The PPA
published an Invitation to Bid with the reservation that it had the right to reject any bid and to
accept such bid it may deem advantageous to the govt. Seven companies submitted bids. The
Committee recommended that the contract be awarded to Intl. Container Terminal Services on
the ground that it offered the best technical and financial proposal. Secretary Reyes awarded
the contract to ICTSI. Before the contract could be signed, two cases were filed questioning the
legality or regularity of the bidding.
The President approved the proposed MICT contract. The PPA and ICTFSI perfected the
contract. Rodolfo Albano, a member of the House of Representatives filed the present case
assailing the award of the contract on the ground that since the MICT is a public utility, it needs
a legislative franchise before it can legally operate as a public utility.
Issue: WON a legislative franchise is necessary to obtain an authority to operate
Held: Not exactly.
A franchise specially granted by Congress is not necessary for the operation of the MICT
by a private entity. Even if the MICT be considered a public utility or a public service on the theory
that it is a wharf or a dock as contemplated by the Public Service Act, its operation would not
necessarily call for a legislative franchise. Legislative franchises are not required before each and
every public utility may operate. The law has granted certain administrative agencies the power
to grant licenses for or to authorize the operation of certain public utilities [1. LTFRB Certificates
of Public Convenience authorizing the operation of public land transportation services provided
by motorized vehicles; 2. ERB operation of electric power utilities and services except electric
coops]
Reading EO 30 and PD 857 together, the PPA has been empowered to undertake by it or
to authorize the operation and mgmt. of the MICT by another by contract. The latter power
having been delegated to the PPA, a legislative franchise is no longer necessary. In this case, the
PPA contracting with ICTSI is wholly within its jurisdiction and powers.
Tatad v. Garcia, 243 SCRA 436 (1995)
TATAD v GARCIA
In 1989, the government planned to build a railway transit line along EDSA. No bidding was
made but certain corporations were invited to prequalify. The only corporation to qualify was
the EDSA LRT Consortium which was obviously formed for this particular undertaking. An
agreement was then made between the government, through the Department of
Transportation and Communication (DOTC), and EDSA LRT Consortium. The agreement was
based on the Build-Operate-Transfer scheme provided for by law (RA 6957, amended by RA
7718). Under the agreement, EDSA LRT Consortium shall build the facilities, i.e., railways, and
shall supply the train cabs. Every phase that is completed shall be turned over to the DOTC and
the latter shall pay rent for the same for 25 years. By the end of 25 years, it was projected that
the government shall have fully paid EDSA LRT Consortium. Thereafter, EDSA LRT Consortium
shall sell the facilities to the government for $1.00.

However, Senators Francisco Tatad, John Osmea, and Rodolfo Biazon opposed the
implementation of said agreement as they averred that EDSA LRT Consortium is a foreign
corporation as it was organized under Hongkong laws; that as such, it cannot own a public utility
such as the EDSA railway transit because this falls under the nationalized areas of activities.
ISSUE: W/N the EDSA LRT Consortium may be allowed to operate the public utility
HELD: No. The SC ruled that EDSA LRT Consortium, under the agreement, does not and will not
become the owner of a public utility. Hence, the question of its nationality is misplaced. It is true
that a foreign corporation cannot own a public utility but in this case what EDSA LRT Consortium
will be owning are the facilities that it will be building for the EDSA railway project. There is no
prohibition against a foreign corporation to own facilities used for a public utility.
Telecom v. COMELEC, 289 SCRA 337 (1998)
TELECOM vs. COMELEC
Petitioners challenge the validity of 92 of BP881 (Omnibus Election Code of the
Philippines) on the ground (1) that it takes property without due process of law and
without just compensation; and(2) that it denies radio and television broadcast
companies the equal protection of the laws;
o Section 92 provides that the COMELEC shall procure radio and television time
whichshall be allocated equally and impartially among the candidates. This is
free of charge.
Petitioners contend that their air time is taken without payment of just compensation.
The primary source of revenue is the sale of air time to advertisers.

ISSUE: Whether or not Section 92 of B.P. No. 881 constitutes taking of property without due
process of law and without just compensation.
RULING: All broadcasting stations is licensed by the government. This is, therefore, a franchise,
which is a privilege subject to amendment by Congress. The Art. XII sec 11 of the Constitution
authorizes the amendment of franchises for the common good. In this case, the common good
involved is for the voters who have the right to be fully informed of the issues in an election.
JG Summit Holdings v. CA, 345 SCRA 143 (2000)
JG SUMMIT vs CA

National Investment and Development Corporation (NIDC) and Kawasaki entered into a
Joint Venture Agreement in a shipyard business named PHILSECO, with a shareholding
of 60-40 respectively. NIDCs interest was later transferred to the National Government.
Through a series of transfers, NIDCs rights in PHILSECO went to the National
Government. It was decided that PHILSECO should be privatized by selling 87.67% of its
total outstanding capital stock to private entities. After negotiations, it was agreed that
Kawasakis right of first refusal under the JVA be exchanged for the right to top by five
percent the highest bid for said shares. Kawasaki said that Philyards Holdings, Inc. (PHI),
in which it was a stockholder, would exercise this right in its stead.
Because of the right to top the highest bid, it was able to top JG Summits bid. JG
Summit protested, contending that PHILSECO is a public utility, the subsequent bidding
granted Kawasaki more than 40% of its ownership.

ISSUE: Whether or not respondents act is valid.


RULING: No. A shipyard such as PHILSECO being a public utility as provided by law, Art XII sec 11
of the constitution applies. Should the NIDC opt to sell its shares of stock to a third party,
Kawasaki could only exercise its right of first refusal to the extent that its total shares of stock
would not exceed 40% of the entire shares of stock of SNS or PHILSECO.
Associated Communications v. NTC, GR No. 144109, February 17, 2003
ASSOCIATED COMMUNICATIONS vs. NTC

On 1994, MOU between broadcasting groups and NTC was established which requires a
congressional franchise for a broadcasting station to operate.
Petition, in the same year, tried to acquire a congressional franchise but failed due to
lack of requirements. However, petitioner was given temporary permit to operate by
the NTC which would expire in 1997.
On 1997, NTC informed that ACWS needs to acquire a congressional franchise before it
can operate. Yet, petitioner failed to gain such franchise. Therefore, NTC held an
administrative case against petitioner and concluded that it its Channel 25 shall be
recalled.
Petitioner raised it to CA and said that it already filed for a congressional franchise and
therefore the recall should be suspended. CA affirmed the decision of NTC.

ISSUE: WON congressional franchise is required before each and every public utility may
operate.
RULING: GR: NO. There is nothing in the law nor in the Constitution which indicates that a
legislative franchise is an indispensable requirement for an entity to operate. HOWEVER, a
legislative franchise will still be required as there is a law (PD 576) which requires for
broadcasting stations to secure a legislative franchise before it can operate.
Eastern Telecom v. Telecom Technologies, GR No. 135992, July 23, 2004
EASTERN TELECOM vs. TELECOM TECHNOLOGIES

NTC granted a Provisional Authority (PA) to Telecommunications Technologies


Philippines, Inc. (TTPI) to install, operate and maintain a local exchange service in some
provinces and the cities of Manila and Caloocan, and the municipality of Navotas.
Before TTPI was able to fully accomplish its rollout obligation, ICC applied for and was
given a PA by the NTC to install, operate and maintain a local exchange service in Manila
and Navotas, two areas which were already covered by TTPI under its PA.
Aggrieved, petitioners filed a petition for review with the CA. Arguing that NTC
committed grave abuse of discretion in granting a provisional authority to respondent
ICC to operate in areas already assigned to TTPI.

ISSUE: Whether or not the CA committed a serious error of law in upholding the Order of the
NTC granting a PA to Respondent to operate LEC services in Manila and Navotas which are
areas already assigned to petitioner TTPI under a prior and subsisting PA.
RULING: NO. The law does not categorically state that the issuance of a PA is exclusive to any
telecommunications company. Neither Congress nor the NTC can grant an exclusive franchise,
certificate, or any other form of authorization to operate a public utility. The Constitution is
quite emphatic that the operation of a public utility shall not be exclusive.
Gamboa v. Teves, 652 SCRA 690
GAMBOA vs. TEVES

This is a petition to nullify the sale of shares of stock of Philippine Telecommunications


Investment Corporation (PTIC) by the government of the Republic of the Philippines,
acting through the Inter-Agency Privatization Council (IPC), to Metro Pacific Assets
Holdings, Inc. (MPAH), an affiliate of First Pacific Company Limited (First Pacific), a Hong
Kong-based investment management and holding company and a shareholder of the
Philippine Long Distance Telephone Company (PLDT).
The petitioner questioned the sale on the ground that it also involved an indirect sale of
12 million shares (or about 6.3 percent of the outstanding common shares) of PLDT
owned by PTIC to First Pacific. With the this sale, First Pacifics common shareholdings in
PLDT increased from 30.7 percent to 37 percent, thereby increasing the total common
shareholdings of foreigners in PLDT to about 81.47%. This, according to the petitioner,
violates Section 11, Article XII of the 1987 Philippine Constitution which limits foreign
ownership of the capital of a public utility to not more than 40%.

ISSUE: Does the term capital in Section 11, Article XII of the Constitution refer to the total
common shares only, or to the total outstanding capital stock (combined total of common and
non-voting preferred shares) of PLDT, a public utility?
RULING: Considering that common shares have voting rights which translate to control, as
opposed to preferred shares which usually have no voting rights, the term capital in Section
11, Article XII of the Constitution refers only to common shares. However, if the preferred
shares also have the right to vote in the election of directors, then the term capital shall
include such preferred shares because the right to participate in the control or management of
the corporation is exercised through the right to vote in the election of directors. In short, the
term capital in Section 11, Article XII of the Constitution refers only to shares of stock that
can vote in the election of directors.
PAGCOR v. BIR, 645 SCRA 338
PAGCOR v. BIR, 645 SCRA 338
Facts:

PAGCOR was created pursuant to a Presidential Decree and was exempted from the
payment of any type of tax, except a franchise tax. Subsequently, R.A. No. 933710 was
enacted and certain sections of the NIRC of 1997 were amended. The particular
amendment that is at issue in this case is Section 1 of R.A. No. 9337, which amended
Section 27 (c) of the NIRC of 1997 by excluding PAGCOR from the enumeration of GOCCs
that are exempt from payment of corporate income tax.
Issue: whether PAGCOR is still exempt from corporate income tax and VAT with the enactment
of R.A. No. 9337?
Ruling: NO. Article XII, Section 11, of the 1987 Constitution is explicit that no franchise for the
operation of a public utility shall be granted except under the condition that such privilege shall
be subject to amendment, alteration or repeal by Congress as and when the common good so
requires.
PAGCOR was granted a franchise to operate and maintain gambling casinos, clubs and other
recreation or amusement places, sports, gaming pools. Under Section 11, Article XII of the
Constitution, PAGCORs franchise is subject to amendment, alteration or repeal by Congress such
as the amendment under Section 1 of R.A. No. 9377. Hence, the provision in Section 1 of R.A. No.
9337, amending Section 27 (c) of R.A. No. 8424 by withdrawing the exemption of PAGCOR from
corporate income tax, which may affect any benefits to PAGCORs transactions with private
parties, is not violative of the non-impairment clause of the Constitution.
Hontiveros-Baraquel v. Toll Regulatory Board, GR No. 181293, 2015
Hontiveros-Baraquel v. Toll Regulatory Board, G.R. No. 181293, February 23, 2015
Facts:

The Philippine National Construction Corporation (PNCC), pursuant to P.D. 1113 with the
right, privilege, and authority to construct, and operate toll facilities Toll expressways, in
a series of agreements transferred authority to perform operations of the South Metro
Manila Highway to Skyway O & M Corporation (SOMCO). The Legislators and the Union
of PNCC oppose the said transfer. They argue that the Toll & operation Certificate issued
by the the Toll Regulatory board (TRB) to SOMCO is highly irregular and that the transfer
of authority is grossly disadvantageous to the government.
One of the contention of the Petitioners is that SOMCO is not qualified to operate a toll
facility, because it does not meet the nationality requirement for a corporation. They
contend that 40% of SOMCO is owned by CMMTC, a foreign company.
Issue: Whether SOMCO is not qualified to operate because it does not meet the nationality
requirement?
Ruling: NO. Section 11, Article XII of the Constitution provides that "[n]o franchise, certificate, or
any other form of authorization for the operation of a public utility shall be granted except to
citizens of the Philippines or to corporations or associations organized under the laws of the
Philippines at least sixty per centum of whose capital is owned by such citizens x x x." Clearly,
under the Constitution, a corporation at least 60% of whose capital is owned by Filipinos is of
Philippine nationality. Considering this constitutional provision, petitioners' silence on the
ownership of the remaining 60% of the corporations cited is very telling.
Petitioners have not shown how SOMCO fails to meet the nationality requirement for a public
utility operator. It is axiomatic that one who alleges a fact has the burden of proving it. On this
matter, the court found that petitioners have failed to prove their allegation that SOMCO is not
qualified to operate a toll facility for failure to meet the nationality requirement under the
Constitution.
Section 12. Filipino First Policy
Tanada v. Angara, 272 SCRA 18 (1997)
Tanada v. Angara, 272 SCRA 18 (1997)
Facts:

This is a petition seeking to nullify the Philippine ratification of the World Trade
Organization (WTO) Agreement. Wherein the WTO opens access to foreign markets,
especially its major trading partners, through the reduction of tariffs on its exports,
particularly agricultural and industrial products. Thus, provides new opportunities for the
service sector cost and uncertainty associated with exporting and more investment in the
country. These are the predicted benefits as reflected in the agreement and as viewed by
the signatory Senators, a free market espoused by WTO.
It is petitioners position however, that the foregoing national treatment and parity
provisions of the WTO Agreement place nationals and products of member countries on
the same footing as Filipinos and local products, in contravention of the Filipino First
policy of the Constitution.
Issue: Whether the WTO impairs the Filipino First Policy?
Ruling: No. while the Constitution indeed mandates a bias in favor of Filipino goods, services,
labor and enterprises, at the same time, it recognizes the need for business exchange with the
rest of the world on the bases of equality and reciprocity and limits protection of Filipino
enterprises only against foreign competition and trade practices that are unfair. While the
Constitution does not encourage the unlimited entry of foreign goods, services and investments
into the country, it does not prohibit them either. In fact, it allows an exchange on the basis of
equality and reciprocity, frowning only on foreign competition that is unfair.
Section 16. Corporations
NDC v. PVB, 192 SCRA 257 (1990)
NDC v. PVB, 192 SCRA 257 (1990)
Facts:
The Agrix Marketing, Inc. (AGRIX) had executed in favor of private respondent Philippine
Veterans Bank a real estate mortgage over three (3) parcels of land. During the existence
of the mortgage, AGRIX went bankrupt. Hence, PD No. 1717 was enacted by President
Marcos which ordered the rehabilitation of the Agrix Group of Companies to be
administered mainly by the National Development Company.
Section 4(1) of the decree, quoted above, extinguishing all mortgages and other liens
attaching to the assets of AGRIX. This was questioned by the PVB and claims that the
decree impairs the obligation of the contract between AGRIX and the private respondent
without justification.
Issue: Whether the decree impairs the obligation of contract?
Ruling: YES. PD 1717 is an invalid exercise of the police power, not being in conformity with the
traditional requirements of a lawful subject and a lawful method. The extinction of the mortgage
and other liens and of the interest and other charges pertaining to the legitimate creditors of
AGRIX constitutes taking without due process of law, and this is compounded by the reduction
of the secured creditors to the category of unsecured creditors in violation of the equal
protection clause.
Moreover, the new corporation, being neither owned nor controlled by the Government, should
have been created only by general and not special law. And insofar as the decree also interferes
with purely private agreements without any demonstrated connection with the public interest,
there is likewise an impairment of the obligation of the contract.
Section 17. Temporary Take-Over
Agan v. PIATCO, 420 SCRA 575
Agan v. PIATCO, 420 SCRA 575
Facts:

The Government, through then DOTC and PIATCO signed the "Concession Agreement for
the Build-Operate-and-Transfer (BOT) Arrangement of the Ninoy Aquino International
Airport Passenger Terminal III".
The Government granted PIATCO the franchise to operate and maintain the said terminal
during the concession period and to collect the fees, rentals and other charges in
accordance with the rates or schedules stipulated in the agreement. The Agreement
provided that the concession period shall be for twenty-five (25) years.
The agreement provides that in times of emergency, the government may immediately
take over the operations of the Terminal and/or the Terminal Complex. Which was
opposed by Piatco.
Issue: Whether the Temporary Take-over of operations by the government is valid?
Ruling: YES. Article XII, section 17 of the 1987 Constitution envisions a situation wherein the
exigencies of the times necessitate the government to temporarily take over or direct the
operation of any privately owned public utility or business affected with public interest. It is the
welfare and interest of the public which is the paramount consideration in determining whether
or not to temporarily take over a particular business.
However, the concessionaire shall be entitled to reasonable compensation for the duration of
the temporary take over by GRP, which compensation shall take into account the reasonable cost
for the use of the Terminal and/or Terminal Complex, any loss or damage to the Development
Facility, and other consequential damages.
David v. Macapagal-Arroyo, GR No. 171396, May 2006
David v. Macapagal-Arroyo, GR No. 171396, May 2006
"In cases involving liberty, the scales of justice should weigh heavily against government and
in favor of the poor, the oppressed, the marginalized, the dispossessed and the weak
FACTS: As the nation celebrated the 20th Anniversary of the Edsa People Power I, President
Arroyo issued PP 1017 declaring a state of national emergency; elements in the political opposition
have conspired with authoritarians of the extreme Left, represented by the NDF-CPP-NPA and the
extreme Right, represented by military adventurists in a plot to unseat or assassinate President
Arroyo. They considered the aim to oust or assassinate the President and take-over the reigns of
government as a clear and present danger

The import of this provision is that President Arroyo, during the state of national emergency under
PP 1017, can call the military not only to enforce obedience "to all the laws and to all decrees x x
x" but also to act pursuant to the provision of Section 17, Article XII which reads:

Sec. 17. In times of national emergency, when the public interest so requires, the State may, during
the emergency and under reasonable terms prescribed by it, temporarily take over or direct the
operation of any privately-owned public utility or business affected with public interest.

What could be the reason of President Arroyo in invoking the above provision when she issued PP
1017?

The answer is simple. During the existence of the state of national emergency, PP 1017 purports
to grant the President, without any authority or delegation from Congress, to take over or direct
the operation of any privately-owned public utility or business affected with public interest.

ISSUE: Petitioners, particularly the members of the House of Representatives, claim that President
Arroyos inclusion of Section 17, Article XII in PP 1017 is an encroachment on the legislatures
emergency powers.
HELD: This Court rules that such Proclamation does not authorize her during the emergency to
temporarily take over or direct the operation of any privately owned public utility or business
affected with public interest without authority from Congress.
A distinction must be drawn between the Presidents authority to declare "a state of national
emergency" and to exercise emergency powers. To the first, as elucidated by the Court, Section
18, Article VII grants the President such power, hence, no legitimate constitutional objection can
be raised. But to the second, manifold constitutional issues arise.
Section 17, Article XII must be understood as an aspect of the emergency powers clause. The
taking over of private business affected with public interest is just another facet of the emergency
powers generally reposed upon Congress. Thus, when Section 17 states that the "the State may,
during the emergency and under reasonable terms prescribed by it, temporarily take over
or direct the operation of any privately owned public utility or business affected with public
interest," it refers to Congress, not the President. Now, whether or not the President may exercise
such power is dependent on whether Congress may delegate it to him pursuant to a law prescribing
the reasonable terms thereof.
Section 18. Nationalization
Republic v. PLDT, 26 SCRA 620 (1968)
Republic v. PLDT, 26 SCRA 620 (1968)

FACTS: PLDT, and the RCA Communications, Inc., (foreign corp.) entered into an agreement
whereby telephone messages, coming from the United States and received by RCA's domestic
station, could automatically be transferred to the lines of PLDT; and vice-versa, for calls collected
by the PLDT for transmission from the Philippines to the United States.

Soon after its creation in 1947, the Bureau of Telecommunications set up its own Government
Telephone System by utilizing its own appropriation and equipment and by renting trunk lines of
the PLDT to enable government offices to call private parties.

Its application for the use of these trunk lines was in the usual form of applications for telephone
service, containing a statement, above the signature of the applicant, that the latter will abide by
the rules and regulations of the PLDT which are on file with the Public Service Commission. One of
the many rules prohibits the public use of the service furnished the telephone subscriber for his
private use. The Bureau has extended its services to the general public since 1948, using the same
trunk lines owned by, and rented from, the PLDT, and prescribing its (the Bureau's) own schedule
of rates. Through these trunk lines, a Government Telephone System (GTS) subscriber could make
a call to a PLDT subscriber in the same way that the latter could make a call to the former.
The Director of Telecommunications, entered into an agreement with RCA Communications, Inc.,
for a joint overseas telephone service whereby the Bureau would convey radio-telephone
overseas calls received by RCA's station to and from local residents. PLDT complained that the
Bureau was violating the conditions for using the trunk lines not only for the use of government
offices but even to serve private persons or the general public. PLDT gave a notice that if violations
were not stopped, PLDT would sever the connections -which PLDT did.

Republic sued PLDT commanding PLDT to execute a contract, through the Bureau, for the use of
the facilities of defendant's telephone system throughout the Philippines under such terms and
conditions as the court finds it reasonable.

Issue:
Whether or not Republic can command PLDT to execute the contract.
[Under Section 18, may the state compel a public utility to render service in the public interest?]
generally yes, provided just compensation is paid therefor.

Held:
No. The Bureau was created in pursuance of a state policy reorganizing the government offices to
meet the exigencies attendant upon the establishment of a free Gov't of the Phil.

When the Bureau subscribed to the trunk lines, defendant knew or should have known that their
use by the subscriber was more or less public and all embracing in nature.

The acceptance by the defendant of the payment of rentals, despite its knowledge that the
plaintiff had extended the use of the trunk lines to commercial purposes, implies assent by the
defendant to such extended use. Since this relationship has been maintained for a long time and
the public has patronized both telephone systems, and their interconnection is to the public
convenience, it is too late for the defendant to claim misuse of its facilities, and it is not now at
liberty to unilaterally sever the physical connection of the trunk lines.

To uphold PLDT's contention is to subordinate the needs of the general public.


x x x while the Republic may not compel the PLDT to celebrate a contract with it, the Republic
may, in the exercise of the sovereign power of eminent domain, require the telephone company
to permit interconnection of the government telephone system and that of the PLDT, as the needs
of the government service may require, subject to the payment of just compensation to be
determined by the court.
PLDT v. NTC, 190 SCRA 717 (1990)
PLDT v. NTC, 190 SCRA 717 (1990)
FACTS: Private respondent Express Telecommunications Co., Inc. (ETCI) obtained from Congress
Republic Act No. 2090 a franchise to establish radio stations for domestic and transoceanic
telecommunications. Petitioner PLDT invoked the prior operator or protection of investment
doctrine in its opposition to ETCIs subsequent application for Certificate of Public Convenience
and Necessity (CPCN). The National Telecommunications Commission (NTC) granted provisional
authority to ETCI subject to the condition that it shall enter into interconnection agreement
with PLDT. PLDT elevated the case to the Supreme Court pointing out ETCIs defective legislative
franchise to operate telecommunications system, among others. ETCI contends that PLDTs
special civil action must deal only on issues whether the NTC acted without jurisdiction of with
grave abuse of discretion in granting ETCI the assailed provisional authority.
ISSUE: WON PLDT may refuse NTC Order to enter into interconnection agreement with ETCI;
HELD: NO. The PLDT cannot justifiably refuse to interconnect. The interconnection which has
been required of PLDT is a form of intervention with property rights dictated by the
encompassing objective for the common good. The NTC, as the regulatory agency of the State,
merely exercised its delegated authority to regulate the use of telecommunications networks
when it decreed interconnection.
Section 19. Monopolies and Combinations
Energy Regulatory Board v. CA, GR No. 113079, April 20, 2001
Garcia v. Executive Secretary, GR No. 132451, December 17, 1999
Tatad v. Secretary of Energy, 281 SCRA 330
Eastern Assurance v. LTFRB, GR No. 149717, Oct. 7, 2003
Avon v. Luna, GR No. 153674, December 20, 2006
Artcile XVI. General Provisions
Section 3. Immunity From Suit
Republic v. Feliciano, 148 SCRA 424 (1987)
Metran v. Paredes, 79 Phil 819 (1947-1948)
Metran (Metropolitan Transportation Service) v. Paredes, 79 Phil 819 (1947-1948)

FACTS:
The Labor Organization of Metran brought a suit against Metran.
Metran is a semi-governmental transportation entity.
In behalf of Metran, an oral petition for dismissal of the case was made on the ground
that Metran belongs to the Republic of the Philippines and as such, it cannot be sued
ISSUE:
Whether Metran can be sued
RULING:
NO. Where the government is of the people, by the people, and for the people, such
immunity from suit will only be the reaffirmation of the sovereignty of the people
themselves as represented by the government.
In a republican state, like Philippines, government immunity from suit without its
consent is derived from the will of the people themselves in freely creating a
government "of the people, and for the people"-a representative government through
which they have agreed to exercise the powers and discharge the duties of their
sovereignty for the common good and general welfare. In so agreeing, the citizens have
solemnly undertaken to surrender some of their private rights and interest which were
calculated to conflict with the higher rights and larger interests of the people as a whole,
represented by the government thus established by them all. One of those "higher
rights," based upon those "larger interests" is that government immunity. The members
of the respondent Labor Union themselves are part of the people who have freely that
government and participated in that solemn undertaking. In this sense-and a very real
one it is-they are in effect attempting to use themselves along with the rest of the
people represented by their common government-an anomalous and absurd situation
indeed.

NAC v. Teodoro, 91 Phil 203 (1952)


NAC (National Airports Corporation) v. Teodoro, 91 Phil 203 (1952)
FACTS:
Before the abolition of NAC, the Philippine Airlines paid to NAC fees for landing and
parking on Bacolod Airport.
o These fees are payable to Capitol Subdivision which owned the land used by NAC
as airport.
Capitol Subdivision commenced an action against PAL to recover the amount
PAL filed a third party complaint against NAC, but summons were served upon the Civil
Aeronautics Administration since the NAC was already dissolved.
The Solicitor General filed a motion to dismiss on the ground that (1) the NAC has lost its
juridical personality; and (2) agency of the Republic of the Philippines, unincorporated
and not possessing juridical personality, is incapable of suing and being sued
ISSUE:
Whether the Civil Aeronautics Administration (CAA) may be sued
RULING:
YES. Among the general powers of the CAA is to execute contracts of any kind. The
power to sue and be sued is implied from the power to transact private business.
(Doctrine of Implied Powers)
CAA also has the power to prosecute and defend suits for and against the NAC, having
acquired all the properties and liabilities of the latter.

Mobil Philippines v. Customs Arrastre, 18 SCRA 120 (1966)


Mobil Philippines v. Customs Arrastre, 18 SCRA 120 (1966)

FACTS:
4 cases of rotary drill parts were shipped from abroad consigned to Mobil Philippines.
The shipment was discharged to the custody of the Customs Arrastre Service.
The Customs Arrastre Service later delivered to the broker of the consignee 3 cases only
of the shipment
Mobil Philippines filed a suit against the Customs Arrastre Service and the BOC to
recover the value of the undelivered cases plus damages
Defendants filed a motion to dismiss on the ground that they cannot be sued.
ISSUE:
Whether the Customs Arrastre Service and the BOC can be sued
RULING:
NO. There is implied consent to be sued given by the State when the state enters into an
operation that is essentially a business operation, unless the business operation is
merely incidental to the performance of a governmental function, as for instance,
arrastre service.

PNR v. IAC, 217 SCRA 401 (1993)


PNR (Philippine National Railways) v. IAC, 217 SCRA 401 (1993)
FACTS:
The case arose from a collision of a passenger express train of PNR and a passenger bus
of Baliwag Transit.
The lower court ruled against PNR.
o It held that PNR is guilty of negligence and civilly liable for damages
ISSUE:
Whether PNR enjoys immunity from suit
RULING:
NO. Even though the laws creating the PNR were mute on its power to sue and be sued,
such prerogative was implied from its general power to transact private business.
(Doctrine of Implied Powers)
The PNR was created not to discharge governmental function but to operate a transport
service which is essentially a business concern, thus barred from invoking immunity
from suit.
Not all government entities, whether corporate or noncorporate, are immune from
suits. Immunity from suit is determined by the character of the objects for which the
entity is organized.
Suits against state agencies with relation to matters in which they have assumed to act
in a private capacity/matters partaking more of the nature of ordinary businesses rather
than functions of government, are not regarded as suits against the State.
When the government enters into a commercial business it abandons its sovereign
capacity and is to be treated like any other private corporation

Ministerio v. CFI, 40 SCRA 464 (1971)


Ministerio v. CFI, 40 SCRA 464 (1971)
FACTS:
Petitioners filed a complaint seeking payment of just compensation for a lot alleging
that the National Government took physical and material possession of it used for
widening of the Gorordo Avenue in Cebu City, without paying just compensation and
without any agreement.
ISSUE:
Whether petitioners could sue the Public Highway Commissioner and Auditor General
without violating the principle of government immunity from suit without its consent
RULING:
NO. The doctrine of non-suability is still applicable even if the defendants named were
public officials since the litigation would result in a financial responsibility of the
government. The liability of the official sued is not personal. The suit is regarded as one
against the government itself.
If the property can no longer be restored and is in fact being enjoyed by the State, then
the State must be deemed to have submitted to the jurisdiction of the court for
purposes of fixing the just compensation.
Municipality of San Fernando v. Firme, 195 SCRA 692
Municipality of San Fernando v. Firme
Facts:
A dump truck of the municipality, while on its way to get sand and gravel for the repair of San
Fernandos municipal streets, collided with a passenger jeepney, resulting to the death and
physical injuries of the passengers. In the complaint for damages, the driver of the dump truck
and Municipality were impleaded. Municipality contended that it cannot be sued.
Issue:
Is the Municipality covered by the immunity from suit?
Held:
YES. The test of liability of the municipality depends on whether or not the driver, acting in
behalf of the municipality, is performing governmental or proprietary functions.
In the case at bar, the driver of the dump truck of the municipality insists that he was on his way
to the Naguilian river to get a load of sand and gravel for the repair of San Fernandos municipal
streets. In the absence of any evidence to the contrary, the regularity of the performance of
official duty is presumed.
Lansang v. CA, GR No. 102667
Lansang v. CA
Fact:
General Assembly of the Blind (GABI) were allegedly awarded a verbal contract of lease in Rizal
Park by the National Parks Development Committee (NPDC).
With the change of government, the new NPDC Chairman sought to clean up Rizal Park and
terminated the said verbal agreement.
GABI filed an action for damages and injunction against the NPDC Chairman, dismissed because
the suit is one that is directed to the State and not merely its official.
Issue:
When is a suit against a public official a suit against the state?
Held:
The rule is that the suit must be regarded as one against the state where satisfaction of the
judgment against the public official concerned will require the state itself to perform a positive
act, such as appropriation of the amount necessary to pay the damages awarded to the
plaintiff.
The rule does not apply where the public official is charged in his official capacity for acts that are
unlawful and injurious to the rights of others. Public officials are not exempt, in their personal
capacity, from liability arising from acts committed in bad faith.
Traders Royal Bank v. IAC, 192 SCRA 305 (1990)
Traders Royal Bank v. IAC
Facts:
Traders Royal Bank extended a loan to the National Media Production Center, an unincorporated
government instrumentality tasked with the function of disseminating government information.
The money was for the purpose of facilitating the broadcast of a basketball season. When the
bank tried to collect, NMPC pleaded immunity from suit.
Issue:
Is NMPC covered by the immunity in this particular instance?
Held:
NO. The immunity from suit may be waived by IMPLIED CONSENT to be sued as when the State
enters into a contract that is proprietary in character. In this case, the contract that is proved to
be proprietary in character.
Republic v. Sandoval, 220 SCRA 124 (1993)
Republic v. Sandoval
Facts:
Farmers marched to Malacanang calling for a genuine land reform program. There was a
confrontation which resulted several casualties including death. As a result, Pres. Aquino issued
AO 11 creating the Citizens Mendiola Commission for the purpose of conducting an
investigation. The most significant recommendation of the Commission was for the heirs of the
deceased and wounded victims to be compensated by the government. Based on
such recommendation, the victims of Mendiola massacre filed an action for damages against the
Republic and the military/police officers involved in the incident.
Issue:
Whether there is a valid waiver of immunity.
Held:
NO. The recommendation made by the Commission to indemnify the heirs of the deceased and
the victims does not in any way mean that liability attaches to the State. AO 11 merely states the
purpose of the creation of the Commission and, therefore, whatever is the finding of the
Commission only serves as the basis for a cause of action in the event any party decides to
litigate the same. Thus, the recommendation of the Commission does not in any way bind the
State.
DA v. NLRC, 227 SCRA 693 (1993)
DA v. NLRC
Facts:
DA and Sultan Security Agency entered into a contract for security services. Guards were
deployed in the premises of the DA. Thereafter, several guards filed a complaint for
underpayment of wages, nonpayment of 13th month pay, uniform allowances, night shift
differential pay, holiday pay, and overtime pay, as well as for damages against the DA and the
security agency. LA ruled that DA and the security agency is solidarily liable.
Issue:
Whether the doctrine of non-suability of the State applies in the case.
Held:
NO. Although the DA has not pretended to have assumed a capacity apart from its being
a governmental entity when it entered into the questioned contract; nor that it could have, in
fact, performed any act proprietary in character. But, be that as it may, the claims of the
complainant security guards clearly constitute money claims. Act 3083 gives the consent of the
State to be sued upon any money claim involving liability arising from contract, express or
implied. Pursuant, however, to Commonwealth Act 327, the money claim must first be brought
to the COA.
EPG Construction v. Vigilar, GR No. 131544, March 16, 2001
EPG Construction v VIGILAR

In 1983, the Ministry of Human Settlement (MHS), through the BLISS Development
Corporation, initiated a housing project on a government property along the east bank of
Manggahan Floodway in Pasig
The MHS entered into a Memorandum of Agreement (MOA) with Ministry of Public
Works and Highways (MPWH) where the latter undertook to develop the housing site and
construct thereon 145 housing units
By virtue of the MOA, MPWH forged individual contracts with petitioners EPG, Ciper,
Septa, Phil. Plumbing, Home Construction, World Builders, Glass World, Performance Builders,
and De Leon Araneta Construction for the construction of the housing units
Under the contracts, the scope of construction and funding covered only around "2/3 of
each housing unit"
Petitioners agreed to undertake and perform "additional constructions" for the
completion of the housing units despite the fact that there was only a verbal promise, and not a
written contract, by the MPWH Undersecretary Aber Canlas that additional funds will be
available and forthcoming
Unpaid balance for the additional constructions amounted to P5,918,315.63
Upon a demand letter from the petitioners, on November 14, 1988, DPWH Asst.
Secretary Madamba opined that payment of petitioners' money claims should be based on
quantum meruit (what one has earned) and should be forwarded to the Commission on Audit
(COA)
In a Letter of the Undersecretary of Budget and Management dated December 20, 1994,
the amount of P5,819,316.00 was then released for the payment of the petitioners' money
claims under Advise of Allotment No. A4-1303-04-41-303
In an indorsement dated December 27, 1995, the COA referred anew the money claims
to the DPWH
In a letter dated August 26, 1996, respondent Secretary Gregorio Vigilar denied the
subject money claims
Petitioners filed before the RTC of QC, Branch 226 a Petition for Mandamus to order the
respondent to pay petitioners their money claims plus damages and attorney's fees.
DPWH Sec Vigilar countered that he cannot be sued as he is an agent of the State, and
therefore immune from suit

ISSUE: Whether or not the State is immune from suit

Held: NO, not in this case.


Secretary Vigilar may not conveniently hide under the States cloak of invincibility against suits
considering that this principle yields to certain settled exceptions, most notable of which is that
the State's immunity cannot serve as an instrument perpetrating injustice
Philrock v. Board of Liquidators, 180 SCRA 171 (1989)
PHILROCK v BOARD OF LIQUIDATORS

Facts:

PHILROCK filed in the RTC of Manila a complaint against the Board of Liquidators, as liquidator of
the defunct REPACOM, for: (1) the replacement of the defective rock pulverizing machinery
purchased from REPACOM, or, as alternative, to refund the purchase at 31% of its contract price;
(2) reparation for losses incurred due to the increased expenses of maintaining the plant at
Php5,000 a month and Php4,000 per day as unrealized profits and exemplary damages; and (3)
Php50,000 attorney fees plus expenses and costs of the suit.
The RTC decided in favor of PHILROCK. The Solicitor General, in behalf of the State, filed a notice
of appeal on the ground that the payment for damages are public funds, hence, exempt from
attachment and execution. Nevertheless, the RTC judge issued a Writ of Execution. Subsequently
the Board of Liquidators filed a petition for certiorari and prohibition in the Court of Appeals
where the Court of Appeals set aside the Writ of Execution by the RTC. Hence, this petition for
review.

Issue: Whether or not the Board of Liquidators, as a government agency without juridical
capacity, may be sued and held liable as litigators of REPACOM.

Ruling:
No. The Board of Liquidators is a government agency, created under E.O. 372 to administer the
assets and pay the liabilities of the defunct REPACOM, thus it has no juridical personality,
separate and distinct from the government, and therefore, as a general rule, suing it is akin to
suing the State. The State enjoys immunity from suit except when it conducts business through
a government-owned and controlled corporation or a non-corporate agency set up primarily
for a business purpose, and even then, the State may not be liable for damages since the purse
of the State, or the disbursement of public funds is in the discretion of the Legislature. The
functions and public services rendered by the State cannot be allowed to be paralyzed or
disrupted by the diversion of public funds from their legitimate specific objectives, as
appropriated by law. Although the liability of REPACOM has been ascertained, the State is at
liberty to determine for itself how to satisfy such liability. Funds should be appropriated by the
Legislature for the specific purpose of satisfying the judgement in favor of PHILROCK before said
judgement may be paid.
Republic of Indonesia v. Vinzon, GR 154705, June 26, 2003
REPUBLIC OF INDONESIA v VINZON

This is a petition for review of the decision made by Court of Appeals in ruling that the Republic
of Indonesia gave its consent to be sued and voluntarily submitted itself to the laws and
jurisdiction of Philippine courts and that petitioners Ambassador Soeratmin and Minister
Counsellor Kasim waived their immunity from suit.

Petitioner, Republic of Indonesia, represented by its Counsellor, Siti Partinah, entered into a
Maintenance Agreement with respondent James Vinzon, sole proprietor of Vinzon Trade and
Services. The equipment covered by the Maintenance Agreement are air conditioning units and
was to take effect for a period of four years.

When Indonesian Minister Counsellor Kasim assumed the position of Chief of Administration,
he allegedly found respondents work and services unsatisfactory and not in compliance with
the standards set in the Maintenance Agreement. Hence, the Indonesian Embassy terminated
the agreement.

The respondent claims that the aforesaid termination was arbitrary and unlawful. Hence, he
filed a complaint against the petitioners which opposed by invoking immunity from suit.

Issues: Whether or not the Republic of Indonesia can invoke the doctrine of sovereign immunity
from suit; Whether or not petitioners Ambassador Soeratmin and Minister Counsellor Kasim
may be sued herein in their private capacities.

Discussions:

The rule that a State may not be sued without its consent is a necessary consequence of the
principles of independence and equality of States. The practical justification for the doctrine of
sovereign immunity is that there can be no legal right against the authority that makes the law
on which the right depends. In the case of foreign States, the rule is derived from the principle
of the sovereign equality of States, as expressed in the maxim par in parem non habet
imperium. All states are sovereign equals and cannot assert jurisdiction over one another.] A
contrary attitude would unduly vex the peace of nations.

The rules of International Law, however, are not unbending or immune to change. The
increasing need of sovereign States to enter into purely commercial activities remotely
connected with the discharge of their governmental functions brought about a new concept of
sovereign immunity. This concept, the restrictive theory, holds that the immunity of the
sovereign is recognized only with regard to public acts or acts jure imperii (public acts of the
government of a state), but not with regard to private acts or acts jure gestionis (the
commercial activities of a state.)
Rulings: The Supreme Court ruled that the republic of Indonesia cannot be deemed to have
waived its immunity to suit. The mere entering into a contract by a foreign state with a
private party cannot be construed as the ultimate test of whether or not it is an act juri
imperii or juri gestionis. Such act is only the start of the inquiry. There is no dispute that the
establishment of a diplomatic mission is an act juri imperii. The state may enter into contracts
with private entities to maintain the premises, furnishings and equipment of the embassy. The
Republic of Indonesia is acting in pursuit of a sovereign activity when it entered into a contract
with the respondent. The maintenance agreement was entered into by the Republic of
Indonesia in the discharge of its governmental functions. It cannot be deemed to have waived
its immunity from suit.
Shell Philippines v. Jalos, GR No. 179918, September 8, 2010
SHELL v JALOS

In 2003, Efren Jalos et al filed a complaint for damages against Shell before the RTC of
Pinamalayan, Oriental Mindoro. Jalos et al claimed that they were all subsistence fishermen
from the coastal barangay of Bansud, Oriental Mindoro whose livelihood was adversely
affected by the construction and operation of Shells natural gas pipeline in the Malampaya
Natural Gas Project.

Shell claims that it cannot be sued without the States consent under the doctrine of state
immunity from suit. Shell said that under Service Contract 38, it served as an agent of the
Philippine government in the development of the Malampaya gas reserves.

ISSUE: May Shell use the doctrine of State immunity from suit?

HELD: NO. Shell is not an agent of the Republic of the Philippines. It is but a service contractor
for the exploration and development of one of the countrys natural gas reserves. It is not
immune from suit and may be sued for claims even without the States consent.

Vigilar v. Aquino, 639 SCRA 772


VIGILAR v AQUINO
The DPWH sent an invitation to bid to Arnulfo Aquino, owner of AD Aquino Construction, for
the construction of a dike in the Porac River in Pampanga. The contract was eventually awarded
to him. Upon completion, Aquino said that 2/3 of the project cost (about 1.2 million) was still
unpaid, but the DPWH refused to pay. Aquino filed a complaint for the collection of a sum of
money with damages before the RTC of Guagua. The DPWH invoked state immunity. The RTC
ruled in favor of Aquino, but the CA reversed.
ISSUE: W/N state immunity can be invoked in this case
HELD: NO. The DPWH cannot escape the obligation to compensate Aquino for services
rendered and work actually done by invoking state immunity from suit. The doctrine of state
immunity cannot be used as an instrument for perpetrating an injustice to a citizen.
ATO v. Ramos, 644 SCRA 36
ATO vs. RAMOS

Sps. Ramos discovered that a portion of their land (somewhere in Baguio) was being
used as part of the runway and running shoulder of the Loakan Airport which is
operated by ATO.
Sometime in 1995, respondents agreed to convey the subject portion by deed of sale to
ATO in consideration of the amount of Php778,150.00. However, ATO failed to pay
despite repeated verbal and written demands.
Thus, an action for collection against ATO was filed by the respondents before the RTC.
ATOs primary contention was that the deed of sale was entered into the performance
of governmental functions. RTC ruled in favor of the respondents. CA affirmed RTC.
Hence, the petition.

ISSUE: Whether ATO can invoke state immunity


RULING: NO. The States immunity from suit does not extend to the petitioner (ATO) because it
is an agency of the State engaged in an enterprise that is far from being the States exclusive
prerogative. The SC further observes that the doctrine of sovereign immunity cannot be
successfully invoked to defeat a valid claim for compensation arising from the taking without
just compensation and without the proper expropriation proceedings being first resorted to of
the plaintiffs property.
Heirs of Diosdado Mendoza v. DPWH, GR No. 203834, 729 SCRA 654, July 9, 2014
HEIRS OF DIOSDADO MENDOZA vs. DPWH

Mendoza, doing business under the name DSuperior Builders, won several construction
biddings under an agriculture project.
Mendozas services, due to infirmities to their construction, were recommended for
forfeiture.
Aggrieved, he brought this to the DPWH which recommended for the rebidding. It also
blacklisted Mendoza from participating in any bidding.
Mendoza brought the matter to the RTC of Manila.

ISSUE: Whether the CA erred in ruling that the DPWH has no juridical personality of its own and
that Mendoza's action was a suit against the State.
RULING: The general rule is that a state may not be sued, but it may be the subject of a suit if it
consents to be sued, either expressly or impliedly. There is express consent when a law so
provides, while there is implied consent when the State enters into a contract or it itself
commences litigation. This Court explained that in order to determine implied waiver when
the State or its agency entered into a contract, there is a need to distinguish whether the
contract was entered into in its governmental or proprietary capacity, when the contract
involves its sovereign or governmental capacity no such waiver may be implied.
The contracts that the DPWH entered into with Mendoza were done in the exercise of its
governmental functions. Thus, the CA correctly ruled that the DPWH enjoys immunity from suit
Hermano Oil Manufacturing & Sugar Corporation v.Toll Regulatory General Considerations
Board, G.R. No. 167290, November 26, 2014
HERMANO OIL vs. TOLL REGULATORY BOARD

Hermano Oil Corporation owned a parcel of land at NLEX. The petitioner requested that
respondent Toll Regulatory Board (TRB) grant an easement of right of way, for it had
been deprived of its enjoyment and possession by the fence that barred its entry. TRB
denied based on the Limited Access Highway Act.
Hence, petitioner sued TRB and Engr. Dumlao demanding specific performance, the
grant of the easement of right of way and damages being deprived of its property
without due process, just compensation and equal protection of the law.

ISSUE: WON respondent TRB, although not strictly a government agency, should enjoy
immunity from suit.
RULING: YES. The TRB, Dumlao and the DPWH correctly invoked the doctrine of sovereign
immunity in their favor. The TRB and the DPWH performed purely or essentially
government or public functions. As such, they were invested with the inherent power of
sovereignty. Being unincorporated agencies or entities of the National Government, they
could not be sued as such.
Section 6. Police Force
Quilonia v. The General Court Martial GR No. 9660, March 4, 1992
QUILONA vs. COURT MARTIAL

Quilona, a policeman, was charged before the General Court Martial for 2 counts of
murder.
At the time of arraignment, the petitioner moved to be tried in a civilian court (by this
time, the PNP Law was already signed but will only take effect the next year). The
arraignment was reset to Dec 28, 1990.
On the said date, the Court Martial denied the motion and insisted on the arraignment
on the same date. Quilona refused to enter a plea, so the respondent court ordered the
entry of not guilty plea.
Petitioner filed this petition for certiorari and prohibition with preliminary injunction
and/or restraining order, alleging that respondent court acted with grave abuse of
discretion in denying his motion for inhibition.

ISSUE: WON respondent court martial acted with grave abuse of discretion amounting to or
excess of jurisdiction in proceeding with the arraignment of the petitioner on 28 December
1990.
RULING: YES. Although Republic Act No. 6975 was not yet in effect when petitioner was
arraigned on 28 December 1990, nevertheless, respondent court martial knew or should have
known that the said Act had already been signed and shall take effect on 1 January 1991. It is
precisely for this reason that respondent court martial decided to have the petitioner's motion
to inhibit argued on 28 December 1990 and thereafter arraigned the petitioner on the same
day despite his vehement refusal to enter a plea.
By closing its eyes to the provisions of Sections 2 and 46, indelicately asserting its military
jurisdiction rather than letting go of the case to civilian jurisdiction to effectuate and give flesh
to the avowed policy and intent of the law, respondent Court committed grave abuse of
discretion.
Carpio v. Executive Secretary 206 SCRA 290 (1992)
Carpio v. Executive Secretary 206 SCRA 290 (1992)
Facts:

Republic Act No. 6975 entitled "An Act Establishing The Philippine National Police Under
A Reorganized Department Of The Interior And Local Government, And For Other
Purposes" was passed and approved by President Aquino. This was questioned by
petitioner on the ground that it emasculated the National Police Commission by limiting
its power "to administrative control" over the Philippine National Police (PNP).
He also contends that Section 12 of the questioned Act constitutes an "encroachment
upon, interference with, and an abdication by the President of, executive control and
commander-in-chief powers.
Issue: Whether RA 6975 is valid?
Ruling: YES. Article XVI, Section 6 thereof, merely mandates the statutory creation of a national
police commission that will administer and control the national police force to be established
thereunder. The circumstance that the NAPOLCOM and the PNP are placed under the
reorganized DILG is merely an administrative realignment that would bolster a system of
coordination and cooperation among the citizenry, local executives and the integrated law
enforcement agencies and public safety agencies created under the assailed Act, the funding of
the PNP being in large part subsidized by the national government.
Department of Budget v. Manilas Finest, GR No. 169466, May 9, 2007
Department of Budget v. Manilas Finest, GR No. 169466, May 9, 2007
Facts:

PD 765 was issued constituting the Integrated National Police (INP) to be composed of
the Philippine Constabulary (PC). Subsequently, RA 6975 or the PNP Law was enacted.
Under Section 23 of said law, the PNP would initially consist of the members of the
Integrated National Police (INP), created under P.D. No. 765, as well as the officers and
enlisted personnel of the PC.
R.A. No. 6975 was amended by R.A. No. 8551 which reengineered the retirement scheme
in the police organization. Relevantly, PNP personnel stood to collect more retirement
benefits than what INP members of equivalent rank, who had retired under the INP Law,
received.
Hence, the INP members filed a petition for declaratory relief. INP members claims that
they are truly absorbed and equally considered as PNP-retirees and thus, entitled to enjoy
the SAME or IDENTICAL retirement benefits being bestowed to PNP-retirees by virtue of
said PNP Law.
Issue: Whether RA 6975 abolish the INP?
Ruling: NO. what the law provided was for the absorption, transfer, and merger of the INP, as
well as other offices comprising of the PC-INC with the PNP. Hence, they are not excluded from
availing themselves of the retirement benefits accorded to the PNP retirees.
Mendoza v. PNP, GR No. 139658, June 21, 2005
Mendoza v. PNP, GR No. 139658, June 21, 2005
Facts:

A affidavit-complaint for illegal arrest, illegal detention, physical injuries, and robbery was
filed against PNP members namely: PO3 William M. Mendoza and PO2 Angelita Ramos by
Teodoro V. Conti. They both denied the charge however was found guilty by the Regional
director.
appeal to the Regional Appellate Board (RAB) of the National Police Commission
(NAPOLCOM), National Capital Region. They went to the RTC to file a motion for
reconsideration.
The respondents claim that their case should be dismissed on the ground of failure to
exhaust the administrative remedy. They should have appealed to the DILG and CSC
before going to the court.
Issue: Whether there was failure to exhaust administrative remedy?
Ruling: YES. Section 6, Article XVI of the Constitution provides that the State shall establish and
maintain one police force which shall be civilian in character. Consequently, the PNP falls under
the civil service pursuant to Section 2(1), Article IX-B, also of the Constitution. Petitioners failure
to exhaust all administrative remedies is fatal to his cause. It is elementary that where, a remedy
is available within the administrative machinery, this should first be resorted to.
Article XVII. Amendments or Revisions
Section 1. Amendment or Revision
RA 6132, Constitutional Convention Act of 1970
Imbong v. COMELEC, 35 SCRA 28 (1970) * Constituent v. Legislative Power
Imbong v. COMELEC, 35 SCRA 28 (1970)
Facts:

Manuel Imbong and Raul Gonzales, filing separate cases and both interested in running
as candidates for delegates to the Constitutional Convention, question the
constitutionality of R.A. No. 6132, claiming that it prejudices their rights as such
candidates.
The Congress, acting as a Constituent Assembly, passed Res. No. 2 which called for a
Constitutional Convention which shall have two delegates from each representative
district. Resolution No. 4 was passed to amend resolution 2 by providing that the
convention shall be composed of 320 delegates with at least two delegates from each
representative district.
The Congress again acting as a legislative body, enacted R.A. 6132, implementing Res Nos.
2 and 4 and expressly repealing R.A 4914 which previously implemented Res. No. 2.
Gonzales assails the validity of Sections 2, 4, 5, and par. 1 of 8(a), and the entire law, while
Imbong questions the constitutionality of par. 1 of Sec. 8(a) of said R.A. 6132
Issue: Does the Congress have the right to call for a constitutional convention and set the
parameters of such convention?
Ruling: YES. The Congress has authority to call a constitutional convention as the constituent
assembly. The Congress also has the authority to enact implementing details, contained in Res.
Nos. 2 and 4 and R.A. 6132, since such details are within the competence of the Congress in
exercise of its legislative power.

Lambino v. COMELEC, 505 SCRA 160 *distinction between Amendment and Revision

Section 2. Initiative
RA No. 6735, An Act Providing for a System of Initiative and Referendum
Defensor-Santiago v. COMELEC, 270 SCRA 106 (1997); MR (1997)
Lambino v. COMELEC, 505 SCRA 160 (2006) *SC declared RA 6735 as sufficient and adequate for
a peoples initiative, effectively abandoning the ruling in Defensor-Santiago v. COMELEC.

Section 3. Constitutional Convention


RA 6132, Constitutional Convention Act of 1970
Tolentino v. COMELEC, GR L-34150, October 16, 1971

Section 4. Ratification
Gonzales v. COMELEC, 21 SCRA 774 (1967)
Tolentino v. COMELEC, 41 SCRA 702 (1971)
Javellana v. ES, GR L-36142, March 31, 1973

Article XVIII Transitory Provisions

Section 26. Ill-Gotten Wealth; Sequestration/Freeze Orders


Cojuangco v. Roxas, 195 SCRA 797 (1991)

Section 27. Effectivity


De Leon v. Esquerra, 152 SCRA 602 (1987)

You might also like